Bell's theorem refuted: Einstein and locality prevail

Foundations of physics and/or philosophy of physics, and in particular, posts on unresolved or controversial issues

Re: Bell's theorem refuted: Einstein and locality prevail

Postby local » Tue Oct 19, 2021 7:17 pm

Gordon Watson wrote:Thanks local, PM your email address, or email me at eprb@me.com, and I'll send you a copy.

Thank you Gordon, but I prefer to protect my anonymity here. Not saying you'd intentionally disclose my identity but sometimes things slip out. I have to protect myself from harassment by aggressive over-the-hill statistics professors. :roll:

HOWEVER, as far as I know, AND BETTER: you only need register FOR FREE to see and PARTICIPATE in the whole discussion, including much squabbling among Bellians Gill, Laidlaw, Lambare, Jackson.

Sounds fun but they require access to my Google contacts, etc. None of their damn business.
local
 
Posts: 295
Joined: Mon Aug 05, 2019 1:19 pm

Re: Bell's theorem refuted: Einstein and locality prevail

Postby local » Tue Oct 19, 2021 7:24 pm

FrediFizzx wrote: Justo, that is a lame copout that you don't know GA. What about quaternions? Don't know those either? If not, then you have no business being in these discussions.

Oh no! That means I'm lame too. How to sleep tonight?

Seems that as long as Gordon doesn't appeal to GA we can at least comment on his flatlander stuff. Justo and I both explicitly disavowed any generalization to GA analyses. Please don't hold that against us. :cry:
Last edited by local on Tue Oct 19, 2021 8:11 pm, edited 2 times in total.
local
 
Posts: 295
Joined: Mon Aug 05, 2019 1:19 pm

Re: Bell's theorem refuted: Einstein and locality prevail

Postby FrediFizzx » Tue Oct 19, 2021 7:47 pm

local wrote:
FrediFizzx wrote: Justo, that is a lame copout that you don't know GA. What about quaternions? Don't know those either? If not, then you have no business being in these discussions.

Oh no! That means I'm lame too. How will I sleep tonight?

Seems that as long as Gordon doesn't appeal to GA we can at least comment on his flat-lander stuff. Justo and I both explicitly disavowed any generalization to GA analyses. Please don't hold that against us. :cry:

What? You don't know quaternions either? I created this forum mainly for the discussion of Joy's models. Now, I have to read thru a bunch of other nonsense when sick. Ugh! Ask questions about what you don't understand about Joy's models and maybe you might learn something or two new. Get with the program you guys!!
.
FrediFizzx
Independent Physics Researcher
 
Posts: 2905
Joined: Tue Mar 19, 2013 7:12 pm
Location: N. California, USA

Re: Bell's theorem refuted: Einstein and locality prevail

Postby local » Tue Oct 19, 2021 7:58 pm

Can work on that but it might be too deep. Or, be careful what you ask for, you might get it. Any traffic is good for a forum so I won't stop posting flatlander stuff. As I said, the forum is a unique resource. Onward and upward!

Sorry to hear you are ill and hope you get better soon, Fred.
local
 
Posts: 295
Joined: Mon Aug 05, 2019 1:19 pm

Re: Bell's theorem refuted: Einstein and locality prevail

Postby gill1109 » Wed Oct 20, 2021 10:53 pm

Gordon Watson wrote:Dear Richard,

Thanks for the gillibluster: extreme verbosity with much avoidance and next-to-zero meaningful content.

Since all my paragraphs and relations are identified by a number:

1. Please identify where my schoolboy math does not acknowledge basic logic.

2. Please identify claims which contradict well known mathematical results.

3. Please show where I am confused by the meaning of words like "or" and "and".

4. The only point of difference that I am aware of is WRT my eqn (5): you see it as a nonlocal relation, whereas I show that it a conditional probability over clearly local (and spacelike separated) results!

So Richard, in happily putting you forward as an adversarial referee, I'll be listing the above (and more) as some of the features the editors may expect from you.

You could here, of course, save some busy people much wasted effort if you would state you case in mathematical terms. At this stage in our discussion, it will be an honest representation when I tell editors that, beyond #4, I have no clue as to what you objections refer to! I might also send them: Against Gill, "Watson's locality proof via tunable sources."

Time you got serious; Gordon

Dear Gordon,

I have stated my case in mathematical terms, many times, and in particular, in many published papers. See Section II of https://arxiv.org/abs/2001.11338, published https://ieeexplore.ieee.org/document/9380450.

Get serious. I’ve read lots of your papers, and commented on them. Now read one page of one of mine and let me know what’s wrong with it.

Regards, Richard

PS I really like the term “gillibluster”. You define it as “extreme verbosity with much avoidance and next-to-zero meaningful content”. What you mean is that for you there is no meaningful content, and that there appears to you to be much avoidance. But I think that that is because of your ignorance of many technical terms and your inability to reason logically. Anyway, it’s always a delight to argue with you and I admire your persistance!

PS also sorry to hear that Fred is not well. Fred, I hope you get well soon!
gill1109
Mathematical Statistician
 
Posts: 2812
Joined: Tue Feb 04, 2014 10:39 pm
Location: Leiden

Re: Bell's theorem refuted: Einstein and locality prevail

Postby FrediFizzx » Wed Oct 20, 2021 11:05 pm

gill1109 wrote: ... I’ve read lots of your papers, and commented on them. Now read one page of one of mine and let me know what’s wrong with it. ...

The Bell inequalities are meaningless no matter which way you try to understand them. It is because of ONE simple fact that you Bell fanatics can't seem to wrap your mind around. NOTHING can exceed the bound on the inequalities!!!!!!!!!!!! :lol: :lol: :lol:
.
FrediFizzx
Independent Physics Researcher
 
Posts: 2905
Joined: Tue Mar 19, 2013 7:12 pm
Location: N. California, USA

Re: Bell's theorem refuted: Einstein and locality prevail

Postby Gordon Watson » Thu Oct 21, 2021 5:31 am

gill1109 wrote:
Gordon Watson wrote:Dear Richard,

Thanks for the gillibluster: extreme verbosity with much avoidance and next-to-zero meaningful content.

Since all my paragraphs and relations are identified by a number:

1. Please identify where my schoolboy math does not acknowledge basic logic.

2. Please identify claims which contradict well known mathematical results.

3. Please show where I am confused by the meaning of words like "or" and "and".

4. The only point of difference that I am aware of is WRT my eqn (5): you see it as a nonlocal relation, whereas I show that it a conditional probability over clearly local (and spacelike separated) results!

So Richard, in happily putting you forward as an adversarial referee, I'll be listing the above (and more) as some of the features the editors may expect from you.

You could here, of course, save some busy people much wasted effort if you would state you case in mathematical terms. At this stage in our discussion, it will be an honest representation when I tell editors that, beyond #4, I have no clue as to what you objections refer to! I might also send them: Against Gill, "Watson's locality proof via tunable sources."

Time you got serious; Gordon

Dear Gordon,

I have stated my case in mathematical terms, many times, and in particular, in many published papers. See Section II of https://arxiv.org/abs/2001.11338, published https://ieeexplore.ieee.org/document/9380450.

Get serious. I’ve read lots of your papers, and commented on them. Now read one page of one of mine and let me know what’s wrong with it.

Regards, Richard

PS I really like the term “gillibluster”. You define it as “extreme verbosity with much avoidance and next-to-zero meaningful content”. What you mean is that for you there is no meaningful content, and that there appears to you to be much avoidance. But I think that that is because of your ignorance of many technical terms and your inability to reason logically. Anyway, it’s always a delight to argue with you and I admire your persistance!

PS also sorry to hear that Fred is not well. Fred, I hope you get well soon!


Dear Gillibluster,

1. I am not aware of you finding any fault in my prior drafts, but let's proceed on the basis that you did. Then each has been corrected in this latest draft. So now, since I reproduce my old results, you need to find new errors. Again, I am not aware that you have found any.

2. WRT Section II, THE HEART OF THE MATTER, that you referred me too: I find much confusion!

In the language of probability theory, the mathematical core [SIC] of Bell’s original proof of his theorem is the assertion that one cannot find a single probability space on which are defined random variables X and Y taking values in the set {−1, +1}, for all a, b, unit vectors in R3, and such that E(XaYb) = −a·b (1) for all a, b.


In not recognising that his inequality (15) was false, here's what he said: "Thus P(b, c) cannot be stationary at the minimum value (-1 at b = c) and cannot equal the quantum mechanical value (3)." So, when I correct his error, no such impossibility exists. Instead, the correct result falls out, and we see that there is no point drawing conclusions from his defective inequality. (Me going on the identify and explain his error.)

Note also, wrt more confusion via your reference to probability theory. At Academia.com, you claimed that a conditional probability over locally-causal space-like separated results is NONLOCAL. *

3. Further:
The result Xa of measuring Alice’s particle is obtained before the direction b in which Bob has chosen to measure his could possibly be known at Alice’s location. Bob’s setting can have no effect whatsoever on Alice’s outcome. But Bob could measure in any direction, and if Alice were to measure in that same direction, her outcome would be the opposite of Bob’s. This suggests [SIC] that all the outcomes Xa, Yb for all possible directions a, b exist in advance, perhaps as deterministic functions of the chosen directions and of some hidden variables.


It suggests to me NO SUCH nonlocal* NONSENSE. It suggests to me that the paired results are correlated via the pairwise conservation of angular momentum. And on that basis alone, the QM result is derived, and confirmed by experiment.

4. {EDIT]: I see, on p.5, that you have again not answered an old question of mine: counterfactual definiteness ("realism"). Can you please define what you mean by each term? And what are the scare-quotes around realism meant to tell us? Then, on p.7, I find more scare-quotes around local realism. Again, please, what are you signalling?

* NB: there is nothing approaching nonlocality in my locally-causal work.

HTH, wrt your continuing confusion [eg, p.7: My “position” on the metaphysical or philosophical issues has varied over the years and remains open."]: can you now please point to some errors in my draft?

Gordon
.
Gordon Watson
 
Posts: 403
Joined: Wed Apr 30, 2014 4:39 am

Re: Bell's theorem refuted: Einstein and locality prevail

Postby Joy Christian » Thu Oct 21, 2021 6:25 am

gill1109 wrote:
I have stated my case in mathematical terms, many times, and in particular, in many published papers. See Section II of https://arxiv.org/abs/2001.11338, published https://ieeexplore.ieee.org/document/9380450.

It should be noted that the paper Gill has linked above is junk paper. It is full of falsehoods, misrepresentations, diversions, logical fallacies, and elementary mathematical mistakes. I have exposed all of these in my response paper, https://ieeexplore.ieee.org/document/9418997, which is also published in IEEE Access and completely debunks all of Gill's supposed arguments.
.
Joy Christian
Research Physicist
 
Posts: 2793
Joined: Wed Feb 05, 2014 4:49 am
Location: Oxford, United Kingdom

Re: Bell's theorem refuted: Einstein and locality prevail

Postby gill1109 » Thu Oct 21, 2021 10:42 pm

Gordon Watson wrote:
gill1109 wrote:
Gordon Watson wrote:Dear Richard,

Thanks for the gillibluster: extreme verbosity with much avoidance and next-to-zero meaningful content.

Since all my paragraphs and relations are identified by a number:

1. Please identify where my schoolboy math does not acknowledge basic logic.

2. Please identify claims which contradict well known mathematical results.

3. Please show where I am confused by the meaning of words like "or" and "and".

4. The only point of difference that I am aware of is WRT my eqn (5): you see it as a nonlocal relation, whereas I show that it a conditional probability over clearly local (and spacelike separated) results!

So Richard, in happily putting you forward as an adversarial referee, I'll be listing the above (and more) as some of the features the editors may expect from you.

You could here, of course, save some busy people much wasted effort if you would state you case in mathematical terms. At this stage in our discussion, it will be an honest representation when I tell editors that, beyond #4, I have no clue as to what you objections refer to! I might also send them: Against Gill, "Watson's locality proof via tunable sources."

Time you got serious; Gordon

Dear Gordon,

I have stated my case in mathematical terms, many times, and in particular, in many published papers. See Section II of https://arxiv.org/abs/2001.11338, published https://ieeexplore.ieee.org/document/9380450.

Get serious. I’ve read lots of your papers, and commented on them. Now read one page of one of mine and let me know what’s wrong with it.

Regards, Richard

PS I really like the term “gillibluster”. You define it as “extreme verbosity with much avoidance and next-to-zero meaningful content”. What you mean is that for you there is no meaningful content, and that there appears to you to be much avoidance. But I think that that is because of your ignorance of many technical terms and your inability to reason logically. Anyway, it’s always a delight to argue with you and I admire your persistance!

PS also sorry to hear that Fred is not well. Fred, I hope you get well soon!


Dear Gillibluster,

1. I am not aware of you finding any fault in my prior drafts, but let's proceed on the basis that you did. Then each has been corrected in this latest draft. So now, since I reproduce my old results, you need to find new errors. Again, I am not aware that you have found any.

2. WRT Section II, THE HEART OF THE MATTER, that you referred me too: I find much confusion!

In the language of probability theory, the mathematical core [SIC] of Bell’s original proof of his theorem is the assertion that one cannot find a single probability space on which are defined random variables X and Y taking values in the set {−1, +1}, for all a, b, unit vectors in R3, and such that E(XaYb) = −a·b (1) for all a, b.


In not recognising that his inequality (15) was false, here's what he said: "Thus P(b, c) cannot be stationary at the minimum value (-1 at b = c) and cannot equal the quantum mechanical value (3)." So, when I correct his error, no such impossibility exists. Instead, the correct result falls out, and we see that there is no point drawing conclusions from his defective inequality. (Me going on the identify and explain his error.)

Note also, wrt more confusion via your reference to probability theory. At Academia.com, you claimed that a conditional probability over locally-causal space-like separated results is NONLOCAL. *

3. Further:
The result Xa of measuring Alice’s particle is obtained before the direction b in which Bob has chosen to measure his could possibly be known at Alice’s location. Bob’s setting can have no effect whatsoever on Alice’s outcome. But Bob could measure in any direction, and if Alice were to measure in that same direction, her outcome would be the opposite of Bob’s. This suggests [SIC] that all the outcomes Xa, Yb for all possible directions a, b exist in advance, perhaps as deterministic functions of the chosen directions and of some hidden variables.


It suggests to me NO SUCH nonlocal* NONSENSE. It suggests to me that the paired results are correlated via the pairwise conservation of angular momentum. And on that basis alone, the QM result is derived, and confirmed by experiment.

4. {EDIT]: I see, on p.5, that you have again not answered an old question of mine: counterfactual definiteness ("realism"). Can you please define what you mean by each term? And what are the scare-quotes around realism meant to tell us? Then, on p.7, I find more scare-quotes around local realism. Again, please, what are you signalling?

* NB: there is nothing approaching nonlocality in my locally-causal work.

HTH, wrt your continuing confusion [eg, p.7: My “position” on the metaphysical or philosophical issues has varied over the years and remains open."]: can you now please point to some errors in my draft?

Gordon
.

Gordon, so you are confused when I express Bell’s claims in terms of Probability 101? You don’t know the mathematical meaning of the word “random variable”?

My “scare quotes” were not meant to scare anyone but to indicate a definition. They signal: from now on when I use the word “local” this is what I mean by it, mathematically, in the context of this paper.

I am not going to point out any errors in your draft till you’ve identified an error in the text I referred you to. Maybe you need to do some background reading. Refresher course on undergraduate probability and statistics.
gill1109
Mathematical Statistician
 
Posts: 2812
Joined: Tue Feb 04, 2014 10:39 pm
Location: Leiden

Re: Bell's theorem refuted: Einstein and locality prevail

Postby Gordon Watson » Thu Oct 21, 2021 11:07 pm

gill1109 wrote:
Gordon Watson wrote:
gill1109 wrote:
Gordon Watson wrote:Dear Richard,

Thanks for the gillibluster: extreme verbosity with much avoidance and next-to-zero meaningful content.

Since all my paragraphs and relations are identified by a number:

1. Please identify where my schoolboy math does not acknowledge basic logic.

2. Please identify claims which contradict well known mathematical results.

3. Please show where I am confused by the meaning of words like "or" and "and".

4. The only point of difference that I am aware of is WRT my eqn (5): you see it as a nonlocal relation, whereas I show that it a conditional probability over clearly local (and spacelike separated) results!

So Richard, in happily putting you forward as an adversarial referee, I'll be listing the above (and more) as some of the features the editors may expect from you.

You could here, of course, save some busy people much wasted effort if you would state you case in mathematical terms. At this stage in our discussion, it will be an honest representation when I tell editors that, beyond #4, I have no clue as to what you objections refer to! I might also send them: Against Gill, "Watson's locality proof via tunable sources."

Time you got serious; Gordon

Dear Gordon,

I have stated my case in mathematical terms, many times, and in particular, in many published papers. See Section II of https://arxiv.org/abs/2001.11338, published https://ieeexplore.ieee.org/document/9380450.

Get serious. I’ve read lots of your papers, and commented on them. Now read one page of one of mine and let me know what’s wrong with it.

Regards, Richard

PS I really like the term “gillibluster”. You define it as “extreme verbosity with much avoidance and next-to-zero meaningful content”. What you mean is that for you there is no meaningful content, and that there appears to you to be much avoidance. But I think that that is because of your ignorance of many technical terms and your inability to reason logically. Anyway, it’s always a delight to argue with you and I admire your persistance!

PS also sorry to hear that Fred is not well. Fred, I hope you get well soon!


Dear Gillibluster,

1. I am not aware of you finding any fault in my prior drafts, but let's proceed on the basis that you did. Then each has been corrected in this latest draft. So now, since I reproduce my old results, you need to find new errors. Again, I am not aware that you have found any.

2. WRT Section II, THE HEART OF THE MATTER, that you referred me too: I find much confusion!

In the language of probability theory, the mathematical core [SIC] of Bell’s original proof of his theorem is the assertion that one cannot find a single probability space on which are defined random variables X and Y taking values in the set {−1, +1}, for all a, b, unit vectors in R3, and such that E(XaYb) = −a·b (1) for all a, b.


In not recognising that his inequality (15) was false, here's what he said: "Thus P(b, c) cannot be stationary at the minimum value (-1 at b = c) and cannot equal the quantum mechanical value (3)." So, when I correct his error, no such impossibility exists. Instead, the correct result falls out, and we see that there is no point drawing conclusions from his defective inequality. (Me going on the identify and explain his error.)

Note also, wrt more confusion via your reference to probability theory. At Academia.com, you claimed that a conditional probability over locally-causal space-like separated results is NONLOCAL. *

3. Further:
The result Xa of measuring Alice’s particle is obtained before the direction b in which Bob has chosen to measure his could possibly be known at Alice’s location. Bob’s setting can have no effect whatsoever on Alice’s outcome. But Bob could measure in any direction, and if Alice were to measure in that same direction, her outcome would be the opposite of Bob’s. This suggests [SIC] that all the outcomes Xa, Yb for all possible directions a, b exist in advance, perhaps as deterministic functions of the chosen directions and of some hidden variables.


It suggests to me NO SUCH nonlocal* NONSENSE. It suggests to me that the paired results are correlated via the pairwise conservation of angular momentum. And on that basis alone, the QM result is derived, and confirmed by experiment.

4. {EDIT]: I see, on p.5, that you have again not answered an old question of mine: counterfactual definiteness ("realism"). Can you please define what you mean by each term? And what are the scare-quotes around realism meant to tell us? Then, on p.7, I find more scare-quotes around local realism. Again, please, what are you signalling?

* NB: there is nothing approaching nonlocality in my locally-causal work.

HTH, wrt your continuing confusion [eg, p.7: My “position” on the metaphysical or philosophical issues has varied over the years and remains open."]: can you now please point to some errors in my draft?

Gordon
.

Gordon, so you are confused when I express Bell’s claims in terms of Probability 101? You don’t know the mathematical meaning of the word “random variable”?

My “scare quotes” were not meant to scare anyone but to indicate a definition. They signal: from now on when I use the word “local” this is what I mean by it, mathematically, in the context of this paper.

I am not going to point out any errors in your draft till you’ve identified an error in the text I referred you to. Maybe you need to do some background reading. Refresher course on undergraduate probability and statistics.

:lol:
Gordon Watson
 
Posts: 403
Joined: Wed Apr 30, 2014 4:39 am

Re: Bell's theorem refuted: Einstein and locality prevail

Postby gill1109 » Fri Oct 22, 2021 1:42 am

Gordon, please send me an email if you have any more questions about the *mathematics* which I wrote out in Section II of https://ieeexplore.ieee.org/document/9380450. I would be amazed (but also delighted) if anyone found an error.

Your equation (5) is indeed a conditional probability. You don't derive it from any mathematical physical model. You just say it is local. Your papers says that the singlet correlations are local because you say so.

You do not expose any error in Bell's reasoning.

If you want to prove that Bell was wrong you must give us functions A(a, lambda) and B(b, lambda) taking values +/-1 and a probability distribution rho over a space of possible values of a hidden variable lambda such that the mean value of A(a, .)B(b, .) is - a.b for all directions a and b. If you don't like doing math but do like writing computer programs, write code which evaluates A and B and which picks lambda according to some probability distribution. I'll then pick a and b, use your code to create lambda_1, ..., lambda_n, and (again using your code) average the n values of A(a, lambda_i)B(b, lambda_i). If n is large it should start getting close to -a.b

Nobody ever did that yet. If anyone ever did, they would get the Nobel prize and all quantum physics textbooks would have to be rewritten.
gill1109
Mathematical Statistician
 
Posts: 2812
Joined: Tue Feb 04, 2014 10:39 pm
Location: Leiden

Re: Bell's theorem refuted: Einstein and locality prevail

Postby FrediFizzx » Fri Oct 22, 2021 2:01 am

gill1109 wrote:Gordon, please send me an email if you have any more questions about the *mathematics* which I wrote out in Section II of https://ieeexplore.ieee.org/document/9380450. I would be amazed (but also delighted) if anyone found an error. ...

Your eq. (3) is false therefore an error. You have gone and destroyed the conservation of angular momentum!



The Bell inequalities are meaningless no matter which way you try to understand them. It is because of ONE simple fact that you Bell fanatics can't seem to wrap your mind around. NOTHING can exceed the bound on the inequalities!!!!!!!!!!!! :lol: :lol: :lol:
.
FrediFizzx
Independent Physics Researcher
 
Posts: 2905
Joined: Tue Mar 19, 2013 7:12 pm
Location: N. California, USA

Re: Bell's theorem refuted: Einstein and locality prevail

Postby gill1109 » Fri Oct 22, 2021 2:57 am

FrediFizzx wrote:
gill1109 wrote:Gordon, please send me an email if you have any more questions about the *mathematics* which I wrote out in Section II of https://ieeexplore.ieee.org/document/9380450. I would be amazed (but also delighted) if anyone found an error. ...

Your eq. (3) is false therefore an error. You have gone and destroyed the conservation of angular momentum!




The Bell inequalities are meaningless no matter which way you try to understand them. It is because of ONE simple fact that you Bell fanatics can't seem to wrap your mind around. NOTHING can exceed the bound on the inequalities!!!!!!!!!!!! :lol: :lol: :lol:

Fred, Equation (3) is not part of my proof of the mathematical core of Bell's theorem. Gordon should read only the part of my Section II where I derive a Bell inequality. I did not want to discuss Joy's work with him.
Last edited by FrediFizzx on Fri Oct 22, 2021 3:33 am, edited 1 time in total.
Reason: nonsense deleted
gill1109
Mathematical Statistician
 
Posts: 2812
Joined: Tue Feb 04, 2014 10:39 pm
Location: Leiden

Re: Bell's theorem refuted: Einstein and locality prevail

Postby FrediFizzx » Fri Oct 22, 2021 3:11 am

gill1109 wrote:
FrediFizzx wrote:
gill1109 wrote:Gordon, please send me an email if you have any more questions about the *mathematics* which I wrote out in Section II of https://ieeexplore.ieee.org/document/9380450. I would be amazed (but also delighted) if anyone found an error. ...

Your eq. (3) is false therefore an error. You have gone and destroyed the conservation of angular momentum!




The Bell inequalities are meaningless no matter which way you try to understand them. It is because of ONE simple fact that you Bell fanatics can't seem to wrap your mind around. NOTHING can exceed the bound on the inequalities!!!!!!!!!!!! :lol: :lol: :lol:

Fred, Equation (3) is not part of my proof of the mathematical core of Bell's theorem. ...

Blah! Blah! Blah! to your freakin' nonsense. You said, "I would be amazed (but also delighted) if anyone found an error." An error has been found in Sect. II. You've crushed the conservation of angular momentum. The correct equation would be when b = a,



The Bell inequalities are meaningless no matter which way you try to understand them. It is because of ONE simple fact that you Bell fanatics can't seem to wrap your mind around. NOTHING can exceed the bound on/in/of any of the inequalities!!!!!!!!!!!! :lol: :lol: :lol:
.
FrediFizzx
Independent Physics Researcher
 
Posts: 2905
Joined: Tue Mar 19, 2013 7:12 pm
Location: N. California, USA

Re: Bell's theorem refuted: Einstein and locality prevail

Postby gill1109 » Tue Oct 26, 2021 9:48 am

FrediFizzx wrote:
gill1109 wrote:
FrediFizzx wrote:
gill1109 wrote:Gordon, please send me an email if you have any more questions about the *mathematics* which I wrote out in Section II of https://ieeexplore.ieee.org/document/9380450. I would be amazed (but also delighted) if anyone found an error. ...

Your eq. (3) is false therefore an error. You have gone and destroyed the conservation of angular momentum!




The Bell inequalities are meaningless no matter which way you try to understand them. It is because of ONE simple fact that you Bell fanatics can't seem to wrap your mind around. NOTHING can exceed the bound on the inequalities!!!!!!!!!!!! :lol: :lol: :lol:

Fred, Equation (3) is not part of my proof of the mathematical core of Bell's theorem. ...

Blah! Blah! Blah! to your freakin' nonsense. You said, "I would be amazed (but also delighted) if anyone found an error." An error has been found in Sect. II. You've crushed the conservation of angular momentum. The correct equation would be when b = a,



The Bell inequalities are meaningless no matter which way you try to understand them. It is because of ONE simple fact that you Bell fanatics can't seem to wrap your mind around. NOTHING can exceed the bound on/in/of any of the inequalities!!!!!!!!!!!! :lol: :lol: :lol:
Last edited by FrediFizzx on Tue Oct 26, 2021 10:16 am, edited 1 time in total.
Reason: nonsense deleted
gill1109
Mathematical Statistician
 
Posts: 2812
Joined: Tue Feb 04, 2014 10:39 pm
Location: Leiden

Previous

Return to Sci.Physics.Foundations

Who is online

Users browsing this forum: No registered users and 89 guests

cron
CodeCogs - An Open Source Scientific Library